Suppose $lim_{n to infty}a_n=a$, Prove $lim_{nto infty} frac{1}{2^n} sum_{k=0}^n binom{n}{k}a_k=a$.












2












$begingroup$



Suppose $lim_{n to infty}a_n=a$. Prove that $$lim_{nto infty} frac{1}{2^n} sum_{k=0}^n binom{n}{k}a_k=a,.$$




Here is a proof,



Proof: Consider $t_{n,k}=frac{1}{2^n}binom{n}{k}$,$0 leq t_{n,k}= frac{n(n-1)...(n-k+1)}{2^nk(k-1)...1}leqfrac{n^k}{2^n} $ and by squeeze theorem, $lim_{n to infty}t_{n,k}=0$. Also, $sum_{k=0}^n frac{1}{2^n}binom{n}{k}=1$, so then by Silverman–Toeplitz theorem, we have $lim_{nto infty} frac{1}{2^n} sum_{k=0}^n binom{n}{k}a_k=a$.



Is this proof correct? Is there other cool proofs?










share|cite|improve this question











$endgroup$








  • 1




    $begingroup$
    What is topliz?
    $endgroup$
    – Batominovski
    Dec 14 '18 at 3:41






  • 1




    $begingroup$
    Ultimately this is just a consequence of the fact that $$frac 1 {2^n} sum_{k = 0}^n binom{n}{k} = 1$$ while the individual summands tend to zero, so this acts as an averaging operator. You may be interested in the ergodic theorem, which considers a similar class of averages.
    $endgroup$
    – T. Bongers
    Dec 14 '18 at 3:46










  • $begingroup$
    Sorry! It is Silverman–Toeplitz theorem, I miss spelled it :en.wikipedia.org/wiki/Silverman%E2%80%93Toeplitz_theorem
    $endgroup$
    – nafhgood
    Dec 14 '18 at 3:46










  • $begingroup$
    I'm sure many probabilistic proofs could be given given that this question could be cast as a sequence of random variables. You might draw in that attention with the correct tag.
    $endgroup$
    – Robert Wolfe
    Dec 14 '18 at 5:58
















2












$begingroup$



Suppose $lim_{n to infty}a_n=a$. Prove that $$lim_{nto infty} frac{1}{2^n} sum_{k=0}^n binom{n}{k}a_k=a,.$$




Here is a proof,



Proof: Consider $t_{n,k}=frac{1}{2^n}binom{n}{k}$,$0 leq t_{n,k}= frac{n(n-1)...(n-k+1)}{2^nk(k-1)...1}leqfrac{n^k}{2^n} $ and by squeeze theorem, $lim_{n to infty}t_{n,k}=0$. Also, $sum_{k=0}^n frac{1}{2^n}binom{n}{k}=1$, so then by Silverman–Toeplitz theorem, we have $lim_{nto infty} frac{1}{2^n} sum_{k=0}^n binom{n}{k}a_k=a$.



Is this proof correct? Is there other cool proofs?










share|cite|improve this question











$endgroup$








  • 1




    $begingroup$
    What is topliz?
    $endgroup$
    – Batominovski
    Dec 14 '18 at 3:41






  • 1




    $begingroup$
    Ultimately this is just a consequence of the fact that $$frac 1 {2^n} sum_{k = 0}^n binom{n}{k} = 1$$ while the individual summands tend to zero, so this acts as an averaging operator. You may be interested in the ergodic theorem, which considers a similar class of averages.
    $endgroup$
    – T. Bongers
    Dec 14 '18 at 3:46










  • $begingroup$
    Sorry! It is Silverman–Toeplitz theorem, I miss spelled it :en.wikipedia.org/wiki/Silverman%E2%80%93Toeplitz_theorem
    $endgroup$
    – nafhgood
    Dec 14 '18 at 3:46










  • $begingroup$
    I'm sure many probabilistic proofs could be given given that this question could be cast as a sequence of random variables. You might draw in that attention with the correct tag.
    $endgroup$
    – Robert Wolfe
    Dec 14 '18 at 5:58














2












2








2


0



$begingroup$



Suppose $lim_{n to infty}a_n=a$. Prove that $$lim_{nto infty} frac{1}{2^n} sum_{k=0}^n binom{n}{k}a_k=a,.$$




Here is a proof,



Proof: Consider $t_{n,k}=frac{1}{2^n}binom{n}{k}$,$0 leq t_{n,k}= frac{n(n-1)...(n-k+1)}{2^nk(k-1)...1}leqfrac{n^k}{2^n} $ and by squeeze theorem, $lim_{n to infty}t_{n,k}=0$. Also, $sum_{k=0}^n frac{1}{2^n}binom{n}{k}=1$, so then by Silverman–Toeplitz theorem, we have $lim_{nto infty} frac{1}{2^n} sum_{k=0}^n binom{n}{k}a_k=a$.



Is this proof correct? Is there other cool proofs?










share|cite|improve this question











$endgroup$





Suppose $lim_{n to infty}a_n=a$. Prove that $$lim_{nto infty} frac{1}{2^n} sum_{k=0}^n binom{n}{k}a_k=a,.$$




Here is a proof,



Proof: Consider $t_{n,k}=frac{1}{2^n}binom{n}{k}$,$0 leq t_{n,k}= frac{n(n-1)...(n-k+1)}{2^nk(k-1)...1}leqfrac{n^k}{2^n} $ and by squeeze theorem, $lim_{n to infty}t_{n,k}=0$. Also, $sum_{k=0}^n frac{1}{2^n}binom{n}{k}=1$, so then by Silverman–Toeplitz theorem, we have $lim_{nto infty} frac{1}{2^n} sum_{k=0}^n binom{n}{k}a_k=a$.



Is this proof correct? Is there other cool proofs?







real-analysis sequences-and-series limits proof-verification alternative-proof






share|cite|improve this question















share|cite|improve this question













share|cite|improve this question




share|cite|improve this question








edited Dec 14 '18 at 3:45







nafhgood

















asked Dec 14 '18 at 3:33









nafhgoodnafhgood

1,805422




1,805422








  • 1




    $begingroup$
    What is topliz?
    $endgroup$
    – Batominovski
    Dec 14 '18 at 3:41






  • 1




    $begingroup$
    Ultimately this is just a consequence of the fact that $$frac 1 {2^n} sum_{k = 0}^n binom{n}{k} = 1$$ while the individual summands tend to zero, so this acts as an averaging operator. You may be interested in the ergodic theorem, which considers a similar class of averages.
    $endgroup$
    – T. Bongers
    Dec 14 '18 at 3:46










  • $begingroup$
    Sorry! It is Silverman–Toeplitz theorem, I miss spelled it :en.wikipedia.org/wiki/Silverman%E2%80%93Toeplitz_theorem
    $endgroup$
    – nafhgood
    Dec 14 '18 at 3:46










  • $begingroup$
    I'm sure many probabilistic proofs could be given given that this question could be cast as a sequence of random variables. You might draw in that attention with the correct tag.
    $endgroup$
    – Robert Wolfe
    Dec 14 '18 at 5:58














  • 1




    $begingroup$
    What is topliz?
    $endgroup$
    – Batominovski
    Dec 14 '18 at 3:41






  • 1




    $begingroup$
    Ultimately this is just a consequence of the fact that $$frac 1 {2^n} sum_{k = 0}^n binom{n}{k} = 1$$ while the individual summands tend to zero, so this acts as an averaging operator. You may be interested in the ergodic theorem, which considers a similar class of averages.
    $endgroup$
    – T. Bongers
    Dec 14 '18 at 3:46










  • $begingroup$
    Sorry! It is Silverman–Toeplitz theorem, I miss spelled it :en.wikipedia.org/wiki/Silverman%E2%80%93Toeplitz_theorem
    $endgroup$
    – nafhgood
    Dec 14 '18 at 3:46










  • $begingroup$
    I'm sure many probabilistic proofs could be given given that this question could be cast as a sequence of random variables. You might draw in that attention with the correct tag.
    $endgroup$
    – Robert Wolfe
    Dec 14 '18 at 5:58








1




1




$begingroup$
What is topliz?
$endgroup$
– Batominovski
Dec 14 '18 at 3:41




$begingroup$
What is topliz?
$endgroup$
– Batominovski
Dec 14 '18 at 3:41




1




1




$begingroup$
Ultimately this is just a consequence of the fact that $$frac 1 {2^n} sum_{k = 0}^n binom{n}{k} = 1$$ while the individual summands tend to zero, so this acts as an averaging operator. You may be interested in the ergodic theorem, which considers a similar class of averages.
$endgroup$
– T. Bongers
Dec 14 '18 at 3:46




$begingroup$
Ultimately this is just a consequence of the fact that $$frac 1 {2^n} sum_{k = 0}^n binom{n}{k} = 1$$ while the individual summands tend to zero, so this acts as an averaging operator. You may be interested in the ergodic theorem, which considers a similar class of averages.
$endgroup$
– T. Bongers
Dec 14 '18 at 3:46












$begingroup$
Sorry! It is Silverman–Toeplitz theorem, I miss spelled it :en.wikipedia.org/wiki/Silverman%E2%80%93Toeplitz_theorem
$endgroup$
– nafhgood
Dec 14 '18 at 3:46




$begingroup$
Sorry! It is Silverman–Toeplitz theorem, I miss spelled it :en.wikipedia.org/wiki/Silverman%E2%80%93Toeplitz_theorem
$endgroup$
– nafhgood
Dec 14 '18 at 3:46












$begingroup$
I'm sure many probabilistic proofs could be given given that this question could be cast as a sequence of random variables. You might draw in that attention with the correct tag.
$endgroup$
– Robert Wolfe
Dec 14 '18 at 5:58




$begingroup$
I'm sure many probabilistic proofs could be given given that this question could be cast as a sequence of random variables. You might draw in that attention with the correct tag.
$endgroup$
– Robert Wolfe
Dec 14 '18 at 5:58










2 Answers
2






active

oldest

votes


















1












$begingroup$

Say $|a_n| le C$ for each $n$. For $epsilon > 0$, take $N$ s.t. $|a_n-a| < epsilon$ for $n ge N$. Then for $M ge N^2$, $$frac{1}{2^M}sum_{k=0}^M {M choose k}|a_k-a| le frac{1}{2^M}sum_{k=0}^sqrt{M} {M choose k}2C+frac{1}{2^M}sum_{k=sqrt{M}}^M {M choose k}epsilon.$$ So, $limsup_{M to infty} |frac{1}{2^M}sum_{k=0}^M {M choose k} a_k - a| le epsilon$. This holds for each $epsilon > 0$.






share|cite|improve this answer









$endgroup$





















    0












    $begingroup$

    There's a pretty standard exercise in which we are asked to show that if $a_krightarrow a$ then
    $$biggl(frac{1}{n}sum_{k=1}^n a_kbiggr)rightarrow a,.$$
    It's reasonable to suspect that a similar approach there might work here.



    After one starts down this road, you eventually realize that you'll need to show that




    $$lim_{nrightarrowinfty}frac{1}{2^n}sum_{k=0}^Nbinom{n}{k}=0$$




    where $N$ is some possibly-big, but-nevertheless-fixed positive integer. To accomplish this, we notice
    $$sum_{k=0}^Nbinom{n}{k}leqsum_{k=0}^Nfrac{n^k}{k!}=sum_{k=0}^Nfrac{N^k}{k!}left(frac{n}{N}right)^kleqleft(frac{n}{N}right)^Ne^N$$
    at least once $n$ surpasses $N$. This inequality gives the limit we wanted.



    Now we are ready to start the formal presentation. Let $epsilon>0$. Choose $N_1$ in $mathbb{N}$ such that
    $$|a_n-a|<frac{epsilon}{2}$$
    for every $ngeq N_1$. Let $M$ satisfy the inequality below for every $n$ in $mathbb{N}$
    $$|a_n-a|<M,.$$
    Choose $N_2$ such that
    $$frac{1}{2^n}sum_{k=0}^{N_1-1}binom{n}{k}<frac{epsilon}{2(M+1)}$$
    for every $ngeq N_2$. We now observe the scratch-paper inequalities that inspired the approach:



    begin{align*}left|biggl(,sum_{k=0}^nfrac{1}{2^n}binom{n}{k}a_kbiggr)-a,right|
    &=left|sum_{k=0}^nfrac{1}{2^n}binom{n}{k}(a_k-a)right|\
    &leq frac{1}{2^n}sum_{k=0}^{N_1-1}binom{n}{k}|a_k-a|+sum_{k=N_1}^nfrac{1}{2^n}binom{n}{k}|a_k-a|
    end{align*}

    for sufficiently large $n$. In particular, if we take $n>max(N_1, N_2)$, we then get
    $$left|biggl(,sum_{k=0}^nfrac{1}{2^n}binom{n}{k}a_kbiggr)-a,right|<epsilon,.$$






    share|cite|improve this answer









    $endgroup$













      Your Answer





      StackExchange.ifUsing("editor", function () {
      return StackExchange.using("mathjaxEditing", function () {
      StackExchange.MarkdownEditor.creationCallbacks.add(function (editor, postfix) {
      StackExchange.mathjaxEditing.prepareWmdForMathJax(editor, postfix, [["$", "$"], ["\\(","\\)"]]);
      });
      });
      }, "mathjax-editing");

      StackExchange.ready(function() {
      var channelOptions = {
      tags: "".split(" "),
      id: "69"
      };
      initTagRenderer("".split(" "), "".split(" "), channelOptions);

      StackExchange.using("externalEditor", function() {
      // Have to fire editor after snippets, if snippets enabled
      if (StackExchange.settings.snippets.snippetsEnabled) {
      StackExchange.using("snippets", function() {
      createEditor();
      });
      }
      else {
      createEditor();
      }
      });

      function createEditor() {
      StackExchange.prepareEditor({
      heartbeatType: 'answer',
      autoActivateHeartbeat: false,
      convertImagesToLinks: true,
      noModals: true,
      showLowRepImageUploadWarning: true,
      reputationToPostImages: 10,
      bindNavPrevention: true,
      postfix: "",
      imageUploader: {
      brandingHtml: "Powered by u003ca class="icon-imgur-white" href="https://imgur.com/"u003eu003c/au003e",
      contentPolicyHtml: "User contributions licensed under u003ca href="https://creativecommons.org/licenses/by-sa/3.0/"u003ecc by-sa 3.0 with attribution requiredu003c/au003e u003ca href="https://stackoverflow.com/legal/content-policy"u003e(content policy)u003c/au003e",
      allowUrls: true
      },
      noCode: true, onDemand: true,
      discardSelector: ".discard-answer"
      ,immediatelyShowMarkdownHelp:true
      });


      }
      });














      draft saved

      draft discarded


















      StackExchange.ready(
      function () {
      StackExchange.openid.initPostLogin('.new-post-login', 'https%3a%2f%2fmath.stackexchange.com%2fquestions%2f3038901%2fsuppose-lim-n-to-inftya-n-a-prove-lim-n-to-infty-frac12n-sum%23new-answer', 'question_page');
      }
      );

      Post as a guest















      Required, but never shown

























      2 Answers
      2






      active

      oldest

      votes








      2 Answers
      2






      active

      oldest

      votes









      active

      oldest

      votes






      active

      oldest

      votes









      1












      $begingroup$

      Say $|a_n| le C$ for each $n$. For $epsilon > 0$, take $N$ s.t. $|a_n-a| < epsilon$ for $n ge N$. Then for $M ge N^2$, $$frac{1}{2^M}sum_{k=0}^M {M choose k}|a_k-a| le frac{1}{2^M}sum_{k=0}^sqrt{M} {M choose k}2C+frac{1}{2^M}sum_{k=sqrt{M}}^M {M choose k}epsilon.$$ So, $limsup_{M to infty} |frac{1}{2^M}sum_{k=0}^M {M choose k} a_k - a| le epsilon$. This holds for each $epsilon > 0$.






      share|cite|improve this answer









      $endgroup$


















        1












        $begingroup$

        Say $|a_n| le C$ for each $n$. For $epsilon > 0$, take $N$ s.t. $|a_n-a| < epsilon$ for $n ge N$. Then for $M ge N^2$, $$frac{1}{2^M}sum_{k=0}^M {M choose k}|a_k-a| le frac{1}{2^M}sum_{k=0}^sqrt{M} {M choose k}2C+frac{1}{2^M}sum_{k=sqrt{M}}^M {M choose k}epsilon.$$ So, $limsup_{M to infty} |frac{1}{2^M}sum_{k=0}^M {M choose k} a_k - a| le epsilon$. This holds for each $epsilon > 0$.






        share|cite|improve this answer









        $endgroup$
















          1












          1








          1





          $begingroup$

          Say $|a_n| le C$ for each $n$. For $epsilon > 0$, take $N$ s.t. $|a_n-a| < epsilon$ for $n ge N$. Then for $M ge N^2$, $$frac{1}{2^M}sum_{k=0}^M {M choose k}|a_k-a| le frac{1}{2^M}sum_{k=0}^sqrt{M} {M choose k}2C+frac{1}{2^M}sum_{k=sqrt{M}}^M {M choose k}epsilon.$$ So, $limsup_{M to infty} |frac{1}{2^M}sum_{k=0}^M {M choose k} a_k - a| le epsilon$. This holds for each $epsilon > 0$.






          share|cite|improve this answer









          $endgroup$



          Say $|a_n| le C$ for each $n$. For $epsilon > 0$, take $N$ s.t. $|a_n-a| < epsilon$ for $n ge N$. Then for $M ge N^2$, $$frac{1}{2^M}sum_{k=0}^M {M choose k}|a_k-a| le frac{1}{2^M}sum_{k=0}^sqrt{M} {M choose k}2C+frac{1}{2^M}sum_{k=sqrt{M}}^M {M choose k}epsilon.$$ So, $limsup_{M to infty} |frac{1}{2^M}sum_{k=0}^M {M choose k} a_k - a| le epsilon$. This holds for each $epsilon > 0$.







          share|cite|improve this answer












          share|cite|improve this answer



          share|cite|improve this answer










          answered Dec 14 '18 at 3:41









          mathworker21mathworker21

          8,9421928




          8,9421928























              0












              $begingroup$

              There's a pretty standard exercise in which we are asked to show that if $a_krightarrow a$ then
              $$biggl(frac{1}{n}sum_{k=1}^n a_kbiggr)rightarrow a,.$$
              It's reasonable to suspect that a similar approach there might work here.



              After one starts down this road, you eventually realize that you'll need to show that




              $$lim_{nrightarrowinfty}frac{1}{2^n}sum_{k=0}^Nbinom{n}{k}=0$$




              where $N$ is some possibly-big, but-nevertheless-fixed positive integer. To accomplish this, we notice
              $$sum_{k=0}^Nbinom{n}{k}leqsum_{k=0}^Nfrac{n^k}{k!}=sum_{k=0}^Nfrac{N^k}{k!}left(frac{n}{N}right)^kleqleft(frac{n}{N}right)^Ne^N$$
              at least once $n$ surpasses $N$. This inequality gives the limit we wanted.



              Now we are ready to start the formal presentation. Let $epsilon>0$. Choose $N_1$ in $mathbb{N}$ such that
              $$|a_n-a|<frac{epsilon}{2}$$
              for every $ngeq N_1$. Let $M$ satisfy the inequality below for every $n$ in $mathbb{N}$
              $$|a_n-a|<M,.$$
              Choose $N_2$ such that
              $$frac{1}{2^n}sum_{k=0}^{N_1-1}binom{n}{k}<frac{epsilon}{2(M+1)}$$
              for every $ngeq N_2$. We now observe the scratch-paper inequalities that inspired the approach:



              begin{align*}left|biggl(,sum_{k=0}^nfrac{1}{2^n}binom{n}{k}a_kbiggr)-a,right|
              &=left|sum_{k=0}^nfrac{1}{2^n}binom{n}{k}(a_k-a)right|\
              &leq frac{1}{2^n}sum_{k=0}^{N_1-1}binom{n}{k}|a_k-a|+sum_{k=N_1}^nfrac{1}{2^n}binom{n}{k}|a_k-a|
              end{align*}

              for sufficiently large $n$. In particular, if we take $n>max(N_1, N_2)$, we then get
              $$left|biggl(,sum_{k=0}^nfrac{1}{2^n}binom{n}{k}a_kbiggr)-a,right|<epsilon,.$$






              share|cite|improve this answer









              $endgroup$


















                0












                $begingroup$

                There's a pretty standard exercise in which we are asked to show that if $a_krightarrow a$ then
                $$biggl(frac{1}{n}sum_{k=1}^n a_kbiggr)rightarrow a,.$$
                It's reasonable to suspect that a similar approach there might work here.



                After one starts down this road, you eventually realize that you'll need to show that




                $$lim_{nrightarrowinfty}frac{1}{2^n}sum_{k=0}^Nbinom{n}{k}=0$$




                where $N$ is some possibly-big, but-nevertheless-fixed positive integer. To accomplish this, we notice
                $$sum_{k=0}^Nbinom{n}{k}leqsum_{k=0}^Nfrac{n^k}{k!}=sum_{k=0}^Nfrac{N^k}{k!}left(frac{n}{N}right)^kleqleft(frac{n}{N}right)^Ne^N$$
                at least once $n$ surpasses $N$. This inequality gives the limit we wanted.



                Now we are ready to start the formal presentation. Let $epsilon>0$. Choose $N_1$ in $mathbb{N}$ such that
                $$|a_n-a|<frac{epsilon}{2}$$
                for every $ngeq N_1$. Let $M$ satisfy the inequality below for every $n$ in $mathbb{N}$
                $$|a_n-a|<M,.$$
                Choose $N_2$ such that
                $$frac{1}{2^n}sum_{k=0}^{N_1-1}binom{n}{k}<frac{epsilon}{2(M+1)}$$
                for every $ngeq N_2$. We now observe the scratch-paper inequalities that inspired the approach:



                begin{align*}left|biggl(,sum_{k=0}^nfrac{1}{2^n}binom{n}{k}a_kbiggr)-a,right|
                &=left|sum_{k=0}^nfrac{1}{2^n}binom{n}{k}(a_k-a)right|\
                &leq frac{1}{2^n}sum_{k=0}^{N_1-1}binom{n}{k}|a_k-a|+sum_{k=N_1}^nfrac{1}{2^n}binom{n}{k}|a_k-a|
                end{align*}

                for sufficiently large $n$. In particular, if we take $n>max(N_1, N_2)$, we then get
                $$left|biggl(,sum_{k=0}^nfrac{1}{2^n}binom{n}{k}a_kbiggr)-a,right|<epsilon,.$$






                share|cite|improve this answer









                $endgroup$
















                  0












                  0








                  0





                  $begingroup$

                  There's a pretty standard exercise in which we are asked to show that if $a_krightarrow a$ then
                  $$biggl(frac{1}{n}sum_{k=1}^n a_kbiggr)rightarrow a,.$$
                  It's reasonable to suspect that a similar approach there might work here.



                  After one starts down this road, you eventually realize that you'll need to show that




                  $$lim_{nrightarrowinfty}frac{1}{2^n}sum_{k=0}^Nbinom{n}{k}=0$$




                  where $N$ is some possibly-big, but-nevertheless-fixed positive integer. To accomplish this, we notice
                  $$sum_{k=0}^Nbinom{n}{k}leqsum_{k=0}^Nfrac{n^k}{k!}=sum_{k=0}^Nfrac{N^k}{k!}left(frac{n}{N}right)^kleqleft(frac{n}{N}right)^Ne^N$$
                  at least once $n$ surpasses $N$. This inequality gives the limit we wanted.



                  Now we are ready to start the formal presentation. Let $epsilon>0$. Choose $N_1$ in $mathbb{N}$ such that
                  $$|a_n-a|<frac{epsilon}{2}$$
                  for every $ngeq N_1$. Let $M$ satisfy the inequality below for every $n$ in $mathbb{N}$
                  $$|a_n-a|<M,.$$
                  Choose $N_2$ such that
                  $$frac{1}{2^n}sum_{k=0}^{N_1-1}binom{n}{k}<frac{epsilon}{2(M+1)}$$
                  for every $ngeq N_2$. We now observe the scratch-paper inequalities that inspired the approach:



                  begin{align*}left|biggl(,sum_{k=0}^nfrac{1}{2^n}binom{n}{k}a_kbiggr)-a,right|
                  &=left|sum_{k=0}^nfrac{1}{2^n}binom{n}{k}(a_k-a)right|\
                  &leq frac{1}{2^n}sum_{k=0}^{N_1-1}binom{n}{k}|a_k-a|+sum_{k=N_1}^nfrac{1}{2^n}binom{n}{k}|a_k-a|
                  end{align*}

                  for sufficiently large $n$. In particular, if we take $n>max(N_1, N_2)$, we then get
                  $$left|biggl(,sum_{k=0}^nfrac{1}{2^n}binom{n}{k}a_kbiggr)-a,right|<epsilon,.$$






                  share|cite|improve this answer









                  $endgroup$



                  There's a pretty standard exercise in which we are asked to show that if $a_krightarrow a$ then
                  $$biggl(frac{1}{n}sum_{k=1}^n a_kbiggr)rightarrow a,.$$
                  It's reasonable to suspect that a similar approach there might work here.



                  After one starts down this road, you eventually realize that you'll need to show that




                  $$lim_{nrightarrowinfty}frac{1}{2^n}sum_{k=0}^Nbinom{n}{k}=0$$




                  where $N$ is some possibly-big, but-nevertheless-fixed positive integer. To accomplish this, we notice
                  $$sum_{k=0}^Nbinom{n}{k}leqsum_{k=0}^Nfrac{n^k}{k!}=sum_{k=0}^Nfrac{N^k}{k!}left(frac{n}{N}right)^kleqleft(frac{n}{N}right)^Ne^N$$
                  at least once $n$ surpasses $N$. This inequality gives the limit we wanted.



                  Now we are ready to start the formal presentation. Let $epsilon>0$. Choose $N_1$ in $mathbb{N}$ such that
                  $$|a_n-a|<frac{epsilon}{2}$$
                  for every $ngeq N_1$. Let $M$ satisfy the inequality below for every $n$ in $mathbb{N}$
                  $$|a_n-a|<M,.$$
                  Choose $N_2$ such that
                  $$frac{1}{2^n}sum_{k=0}^{N_1-1}binom{n}{k}<frac{epsilon}{2(M+1)}$$
                  for every $ngeq N_2$. We now observe the scratch-paper inequalities that inspired the approach:



                  begin{align*}left|biggl(,sum_{k=0}^nfrac{1}{2^n}binom{n}{k}a_kbiggr)-a,right|
                  &=left|sum_{k=0}^nfrac{1}{2^n}binom{n}{k}(a_k-a)right|\
                  &leq frac{1}{2^n}sum_{k=0}^{N_1-1}binom{n}{k}|a_k-a|+sum_{k=N_1}^nfrac{1}{2^n}binom{n}{k}|a_k-a|
                  end{align*}

                  for sufficiently large $n$. In particular, if we take $n>max(N_1, N_2)$, we then get
                  $$left|biggl(,sum_{k=0}^nfrac{1}{2^n}binom{n}{k}a_kbiggr)-a,right|<epsilon,.$$







                  share|cite|improve this answer












                  share|cite|improve this answer



                  share|cite|improve this answer










                  answered Dec 14 '18 at 14:35









                  Robert WolfeRobert Wolfe

                  5,79722763




                  5,79722763






























                      draft saved

                      draft discarded




















































                      Thanks for contributing an answer to Mathematics Stack Exchange!


                      • Please be sure to answer the question. Provide details and share your research!

                      But avoid



                      • Asking for help, clarification, or responding to other answers.

                      • Making statements based on opinion; back them up with references or personal experience.


                      Use MathJax to format equations. MathJax reference.


                      To learn more, see our tips on writing great answers.




                      draft saved


                      draft discarded














                      StackExchange.ready(
                      function () {
                      StackExchange.openid.initPostLogin('.new-post-login', 'https%3a%2f%2fmath.stackexchange.com%2fquestions%2f3038901%2fsuppose-lim-n-to-inftya-n-a-prove-lim-n-to-infty-frac12n-sum%23new-answer', 'question_page');
                      }
                      );

                      Post as a guest















                      Required, but never shown





















































                      Required, but never shown














                      Required, but never shown












                      Required, but never shown







                      Required, but never shown

































                      Required, but never shown














                      Required, but never shown












                      Required, but never shown







                      Required, but never shown







                      Popular posts from this blog

                      Quarter-circle Tiles

                      build a pushdown automaton that recognizes the reverse language of a given pushdown automaton?

                      Mont Emei